Schrödinger-Gleichung aus Klein-Gordon-Gleichung?

Man kann QM als 1+0-dimensionale QFT betrachten, Felder sind nur zeitabhängig und heißen daher nur Operatoren, und ich kenne einen Weg, um die Schrödinger-Gleichung von der Klein-Gordon-Gleichung abzuleiten.

Angenommen ein Feld Φ mit geringer Energie E M mit M die Masse des Teilchens, indem definiert wird ϕ wie zum Beispiel Φ ( X , T ) = e ich M T ϕ ( X , T ) und Entwicklung der Gleichung

( 2 + M 2 ) Φ   =   0

Vernachlässigung der T 2 ϕ dann findet man die bekannte Schrödinger-Gleichung:

ich T ϕ   =   Δ 2 M ϕ .

Dennoch bin ich mit dem Wechselfeld nicht ganz zufrieden Wellenfunktion, auch wenn wir annehmen, dass die Anzahl der Teilchen fest ist, und das Feld nun auf einen endlichdimensionalen Hilbert-Raum wirkt (ein Unterteil des vollständigen ersten Fock-Raums für eine bestimmte Anzahl von Teilchen). Hat jemand einen anderen Vorschlag/Argument für diese Ableitung?

Bearbeiten: Als Referenz sind die vorherigen Berechnungen aus Zees Buch QFT in a Nutshell, erste Seite in Kapitel III.5, entnommen. Entsprechend siehe Wikipedia .

David Tong hat eine gute Herleitung/Erklärung dafür in Abschnitt 2.8 hier: damtp.cam.ac.uk/user/tong/qft.html Ich hatte jedoch meine eigenen Zweifel, falls Sie die gleichen Zweifel haben, hier sind Threads, die sie beantworten: physicalforums.com/showthread.php?t=709980 physical.stackexchange.com/q/77290

Antworten (2)

Ich glaube du verwechselst zwei verschiedene Dinge:

  1. Erstens können Sie QM als sehen 0 + 1 (eine zeitliche Dimension) QFT, bei der die Ortsoperatoren (und ihre konjugierten Impulse) im Heisenberg-Bild die Rolle der Felder (und ihrer konjugierten Impulse) in der QFT spielen. Sie können beispielsweise überprüfen, dass die räumliche Rotationssymmetrie in der quantenmechanischen Theorie in eine interne Symmetrie in der QFT übersetzt wird.

  2. Zweitens können Sie die "nicht-relativistische Grenze" (übrigens hässlicher Name, weil die Galileische Relativitätstheorie so relativistisch ist wie die spezielle Relativitätstheorie) der Klein-Gordon- oder Dirac-Theorie nehmen, um eine "nicht-relativistische" Schrödinger-QFT zu erhalten, wo ϕ (in Ihrer Notation) ist ein Quantenfeld anstelle einer Wellenfunktion. Es gibt ein Kapitel in Srednickis Buch, in dem dieses Thema auf einfache und nette Weise angesprochen wird. Dort können Sie auch über Spin-Statistik-Theorem und die Wellenfunktion von Mehrteilchenzuständen lesen. Lassen Sie mich einige Gleichungen hinzufügen, die das hoffentlich verdeutlichen (ich verwende Ihre Notation und natürlich können falsche Faktoren, Einheiten usw. vorkommen):

Das Quantenfeld ist:

ϕ D 3 P A P e ich ( P 2 / ( 2 M ) T P X )

Der Hamiltonoperator ist:

H ich D 3 X ( ϕ T ϕ 1 2 M ich ϕ ich ϕ ) D 3 P P 2 2 M A P A P

Die Entwicklung des Quantenfeldes ist gegeben durch:

ich T ϕ [ ϕ , H ] 2 ϕ 2 M

1-Teilchen-Zustände sind gegeben durch:

| 1 P D 3 P F ~ ( T , P ) A P | 0

(analog kann man Mehrteilchenzustände definieren)

Dieser Zustand verifiziert die Schrödinger-Gleichung:

H | 1 P = ich T | 1 P
iff

ich T F ( T , X ) 2 F ( T , X ) 2 M

Wo F ( T , X ) ist die räumliche Fourier-Transformation von F ~ ( T , P ) .

F ( T , X ) ist eine Wellenfunktion, während ϕ ( T , X ) ist ein Quantenfeld.

Dies ist die freie Theorie, man kann auf ähnliche Weise Interaktion hinzufügen.

Danke für das Update, aber ich bin speziell unter einer Grenzoperation, die mich zu einem "ersten Quantisierungsschema" von einem "zweiten Quantisierungsschema" führen würde, alias, reicht es aus, dass ich eine Schrödinger-Gleichung wiederherstellen und dann konstruieren kann ein konservierter Wahrscheinlichkeitsstrom mit einer positiven Dichtekomponente ( J 0 ) das Feld als Wellenfunktion umzuinterpretieren, deren Betragsquadrat Wahrscheinlichkeiten angibt?
Ich bin mir nicht sicher, wonach Sie suchen. F ist eine Wellenfunktion, die die Schr verifiziert. Gleichung. Der Erwartungswert von ϕ ist auch eine Funktion, die den Schr verifiziert. Gleichung. Solange Sie sie also normalisieren können, erhalten Sie eine quantenmechanische probabilistische Interpretation. Habe ich Ihre Frage beantwortet?
@drake: Der Erwartungswert von ϕ ist nicht der richtige Weg, um Wellenfunktionen aus dem Feld zu extrahieren. Der richtige Weg ist, den Staat zu betrachten ψ ( X ) ϕ ( X ) Wo ψ ist eine Zahl und ϕ ( X ) ist der nichtrelativistische Körper.
@toot: Der obige Kommentar ist die Entsprechung zwischen nichtrelativistischen Feldern und Wellenfunktionen. Wenn Sie ein nichtrelativistisches Erzeugungsfeld mit einer Funktion beschmieren, erzeugen Sie ein Teilchen mit Wellenfunktion ψ .
Danke. Sie ändern nur meine Schreibweise: Ihre ψ ist mein F . Der Erwartungswert ist eine Lösung der Gleichung, er wird üblicherweise als klassisches Feld bezeichnet.

In dieser Antwort nehmen wir zur Verdeutlichung die richtigen Faktoren von auf Und C in der Berechnung in Kapitel III.5 von Zee's QFT in a Nutshell.

(4) Φ ( X , T )   =   2 M exp ( ich M C 2 T ) ϕ ( X , T ) .

Die freie Klein-Gordon- Lagrange-Dichte ist

(5) L   =   | 1 C T Φ | 2 | Φ | 2 | M C Φ | 2   = ( 4 )   | ( C 2 M T ich C M 2 ) ϕ | 2 2 2 M | ϕ | 2 M C 2 2 | ϕ | 2   =   | C 2 M T ϕ | 2 + ich 2 ( ϕ T ϕ T ϕ ϕ ) 2 2 M | ϕ | 2   ich 2 ( ϕ T ϕ T ϕ ϕ ) 2 2 M | ϕ | 2 F Ö R C     .
Der letzte Ausdruck ist die freie Schrödinger-Lagrange-Dichte (bis zu den Ableitungstermen der gesamten Raumzeit), vgl. zB dieser Phys.SE Beitrag.